How to integrate this function?

Click For Summary
The discussion focuses on integrating the equation $$ \int dx = \int \frac{d\phi}{\phi \sqrt{1 - \phi^2}} $$ to arrive at the solution $$ x = \operatorname{arcsech}{\phi} $$. Users share their attempts using apps like Mathway and WolframAlpha, which confirmed the answer but did not clarify the integration process. Suggestions include using trigonometric substitution, specifically setting $$ \sin(u) = \sqrt{1 - \phi^2} $$ or $$ \phi = \cos u $$ to simplify the algebra. Participants emphasize the importance of tracking signs during integration to avoid errors. The conversation highlights various methods to approach the integration while seeking clarity on the process.
Mutatis
Messages
42
Reaction score
0

Homework Statement


I've got to integrate the following $$ \int dx =\int \frac {d\phi} {\phi \sqrt {1 - \phi²}}. $$

Homework Equations

[/B]
I already know the answer but not how to get it. The answer that I got from solution is ## x = \operatorname {arcsech}{\phi} ##.

The Attempt at a Solution


I've used two apps to help me get this answer. First I did it using Mathway and I got:
73jgcy.jpg
.

Then, WolframAlpha told me that ## x = \operatorname {arcsech}{\phi} ## is equal to:
90536r.jpg
.

After that I took that first equation in the first image and with some algebraic manipulations I got:
2v1n4hw.jpg
.

So I don't know how to procedure beyond here. This was the closest I achieved from the final answer.
 

Attachments

  • 73jgcy.jpg
    73jgcy.jpg
    46 KB · Views: 840
  • 90536r.jpg
    90536r.jpg
    13.8 KB · Views: 879
  • 2v1n4hw.jpg
    2v1n4hw.jpg
    7.2 KB · Views: 766
Physics news on Phys.org
Mutatis said:

Homework Statement


I've got to integrate the following $$ \int dx =\int \frac {d\phi} {\phi \sqrt {1 - \phi²}}. $$

Homework Equations

[/B]
I already know the answer but not how to get it. The answer that I got from solution is ## x = \operatorname {arcsech}{\phi} ##.

The Attempt at a Solution


I've used two apps to help me get this answer. First I did it using Mathway and I got:
View attachment 227957 .

Then, WolframAlpha told me that ## x = \operatorname {arcsech}{\phi} ## is equal to:
View attachment 227958 .

After that I took that first equation in the first image and with some algebraic manipulations I got:
View attachment 227959 .

So I don't know how to procedure beyond here. This was the closest I achieved from the final answer.
One approach would be to use a trig substitution, with ##\sin(u) = \sqrt{1 - \phi^2}##, and ##\cos(u)du = \frac{-2\phi d\phi}{\sqrt{1 - \phi^2}}##. The result you get probably won't look like the solution you show, but it should be equal to the solution above within a constant.
 
Mutatis said:
I already know the answer but not how to get it. The answer that I got from solution is ## x = \operatorname {arcsech}{\phi} ##.
You dropped a sign somewhere. The derivative of arcsech is the negative of the integrand.

I'd use the substitution ##\phi = \cos u##. It's pretty much the same as Mark's suggestion, but it makes the algebra a little simpler.
 
Thank you guys! I'm going to try to do it again watching the signs and doing what you've said!
 
Question: A clock's minute hand has length 4 and its hour hand has length 3. What is the distance between the tips at the moment when it is increasing most rapidly?(Putnam Exam Question) Answer: Making assumption that both the hands moves at constant angular velocities, the answer is ## \sqrt{7} .## But don't you think this assumption is somewhat doubtful and wrong?

Similar threads

  • · Replies 9 ·
Replies
9
Views
2K
  • · Replies 15 ·
Replies
15
Views
2K
  • · Replies 2 ·
Replies
2
Views
2K
Replies
3
Views
2K
  • · Replies 4 ·
Replies
4
Views
2K
Replies
10
Views
3K
  • · Replies 11 ·
Replies
11
Views
2K
  • · Replies 14 ·
Replies
14
Views
2K
Replies
4
Views
2K
  • · Replies 105 ·
4
Replies
105
Views
6K